Questions tagged [vortex]

The tag has no usage guidance.

Filter by
Sorted by
Tagged with
19 votes
9 answers
6k views

Vortex in liquid collects particles in center

At xmas, I had a cup of tea with some debris at the bottom from the leaves. With less than an inch of tea left, I'd shake the cup to get a little vortex going, then stop shaking and watch it spin. ...
Kricket's user avatar
  • 291
21 votes
4 answers
3k views

How to show that the Coriolis effect is irrelevant for the whirl/vortex in the sink/bathtub?

There is a common myth that water flowing out from a sink should rotate in direction governed by on which hemisphere we are; this is shown false in many household experiments, but how to show it ...
user avatar
23 votes
6 answers
86k views

Can magnetic fields be redirected and focused at one point?

I know that magnetic fields can be redirected, but... given a situation where you have static magnetic field over a large area, and you want to quickly change the magnetic field strength. Is it ...
mugetsu's user avatar
  • 487
21 votes
9 answers
25k views

Why there's a whirl when you drain the bathtub?

At first I thought it's because of Coriolis, but then someone told me that at the bathtub scale that's not the predominant force in this phenomenon.
Dan's user avatar
  • 353
2 votes
1 answer
14k views

What is actually the characteristic length in fluid dynamics description?

In many fluid-dynamics models and quantities (Reynolds number, Strouhal number... generally various turbulent flow descriptions) there is a parameter (usually $L$ or $\ell$) descripted as ...
Victor Pira's user avatar
  • 2,814
11 votes
2 answers
2k views

The optics of vortex (in water): why there is a bright ring?

This picture is from this YouTube video from "Physics Girl", which shows how you can generate vortices in a swimming pool by using a plate: And there she explains the creation of the shaded circle ...
Jokela's user avatar
  • 2,445
19 votes
6 answers
9k views

How can I understand a Vortex Tube and its efficiency?

A Vortex Tube takes a pressurized input stream, most typically of a gas, and creates two output streams with a temperature differential. Apparently, it has been described as a Maxwell's Demon. Both ...
mart's user avatar
  • 2,132
0 votes
1 answer
1k views

Air core Vortex; Physical explanation of the "air Entrainment Hook" at $F_{co}=0.7$

This question was originally asked in the Outlook of this study (page 197)1 $F_{co}$ combined Froude number, [dimensionless], and can be calculated $F_{co}\thickapprox v_D/ \sqrt{gh}$ where $v_D$ is ...
Jokela's user avatar
  • 2,445
17 votes
2 answers
1k views

Why are superfluid vortex lattices stable?

Both (a) neutral superfluids that are externally rotated, and (b) type-II superconductors (i.e. charged superfluids) under applied magnetic fields between the critical fields $h_{c1}$ and $h_{c2}$, ...
tparker's user avatar
  • 47.5k
11 votes
4 answers
5k views

Why does bathwater make a vortex in the plughole?

When I google for this I just get stuff about whether or not the Coriolis effect makes it go clockwise or anticlockwise, but I don't care which direction it turns in. I want to know why it turns at ...
Adrian May's user avatar
6 votes
3 answers
1k views

What is the theoretical justification for a fluid flow's being irrotational?

I am not a fluid dynamicist, and I really just began thinking about this problem as my curiousity drew me into building an answer for the question What really allows airplanes to fly?. It is very ...
Selene Routley's user avatar
5 votes
3 answers
4k views

When is a flow vortex free?

To solve problems in fluid dynamics one states often the assumption that the flow is vortex free i.e. $rot(u) = 0$ It is a basic assumption which is needed for potential flow problems etc.. My ...
Dimitar Ho's user avatar
5 votes
3 answers
3k views

Whirlpools and Tornados

This may not be a great question. But whenever you drain water, a small whirlpool happens, obviously. This got me thinking... Can we model tornados with this effect, would it even be beneficial?
Calc1DropOut's user avatar
3 votes
1 answer
338 views

Derivation of vorticity equation (incompressible flow)

In the following derivation of the vorticity equation, I do not understand how $\nabla \cdot v=0$ implies $\frac{1}{\rho^2}\nabla \rho \times \nabla p=0$. We start with the Euler equation $$\frac{\...
bhoutik's user avatar
  • 113
2 votes
2 answers
268 views

Could galactic rotation be similar to an irrotational vortex?

Could black holes' near light speed rotation cause galaxies to move like an irrotational vortices?
Ryan S.'s user avatar
  • 35
1 vote
1 answer
105 views

Isentropic fluid: cross product of gradients is zero, why?

In the vorticity equation we have the baroclinic term of the form: $$\frac{ {\nabla}\rho}{\rho}\times\frac{ {\nabla}{P} }{\rho}.$$ Why does it go to zero for isentropic flow? I understand that, if the ...
martín canullán's user avatar
141 votes
12 answers
12k views

Why does my tea periodically alternate its rotational speed after stirring? (Link to video below)

I noticed that after stirring, a bubble in the centre of my mug of tea changed the speed it was rotating at periodically. Speeding up, then slowing down, then speeding up again, etc. Almost like when ...
luke's user avatar
  • 1,107
84 votes
1 answer
39k views

Why do chimneys have these spiral "wings"?

While walking around I noticed something very peculiar. Many chimneys had spiral "wings", while others didn't. I came up with two possibilities: The wind circles around the chimney upwards which ...
Adi's user avatar
  • 1,628
32 votes
3 answers
14k views

Emptying a bottle faster by swirl

A moment ago, I was emptying bottles filled with water (2 liters) that are on the terrace of my house. As I did so I remembered something I saw on TV a some time ago (I don't remember when or where or ...
rnrneverdies's user avatar
  • 1,098
17 votes
5 answers
2k views

Can vorticity be destroyed?

I have a professor that is fond of saying that vorticity cannot be destroyed. I see how this is true for inviscid flows, but is this also true for viscous flow? The vorticity equation is shown below ...
OSE's user avatar
  • 1,575
11 votes
1 answer
12k views

What is a Physically Accurate Explanation for the Kutta Condition?

Countless arguments between highly intelligent people have been waged (on this very site in fact) as to exactly how lift can be explained in an experimentally and mathematically rigorous way. Taking ...
Bryson S.'s user avatar
  • 3,886
8 votes
2 answers
2k views

Why water in the sink follow a curved path?

When you fill the sink with water and then allow the water to be drained, the water forms a vortex.. And then it starts to follow a curved path downwards by effects of gravity. Why this phenomena ...
Force's user avatar
  • 558
8 votes
1 answer
6k views

How deep can a whirlpool descend?

My question is based around comparing the physics of a tornado against, what I imagine/assume to be the marine equivalent of a tornado, that is a whirlpool located either at sea, in a tidal region or ...
user avatar
8 votes
1 answer
2k views

Could the phenomenon of vortex bursting be exploited to reduce wake turbulence?

One of the classic stories in the annals of aerospace engineering is the development (and subsequent redesign) of the F-18 and its Leading Edge Extensions (LEX) due to fatigue problems, problems that ...
Bryson S.'s user avatar
  • 3,886
4 votes
1 answer
2k views

Physical intuition for Lamb vector

In many aerodynamical descriptions involving vortex structures (e.g. the Crocco's theorem) there is a useful physical quantity called Lamb vector: $$ \vec{\omega} \times \vec{v} $$ where $\vec{\...
Victor Pira's user avatar
  • 2,814
4 votes
1 answer
3k views

Vortex anti-vortex?

i'm studying Kosterlitz THouless transition and i have a doubt: what is a vortex anti-vortex configuration? Is this thing? or this one I think that they are quite different !
MementoMori's user avatar
4 votes
1 answer
123 views

What role do quantized vortices play from the Superfluid model in Superfluid Vacuum Theory?

When rotating a superfluid (such as $^{4}$He) below its critical temperature, sometimes quantized vortices pop up, depending how fast one rotates this superfluid. In the SVT model, they propose that ...
jambajuice's user avatar
3 votes
1 answer
353 views

Vortex in superfluid?

I'm studying superfluid helium 4. I have studied that the superfluid has a velocity : $ \vec{v}_s = \frac{\hbar}{m} \vec{\nabla} \varphi(\vec{r}) $ $\rightarrow$ $ \nabla \times \vec{v}_s = 0 $. (...
MementoMori's user avatar
3 votes
0 answers
134 views

Cascade vs Inverse Cascade: Energy flow in 2D and 3D turbulence

We know that vortices tend to coalesce in 2D(inverse cascade) as opposed to 3D where they 'break down' into smaller vortices(cascade) ie. energy flows fro larger to smaller scales in 3D and opposite ...
Soumak B.'s user avatar
3 votes
2 answers
3k views

Why is the eye of a cyclone a forced vortex?

As far as I understand, a forced vortex has a velocity profile $u \propto r$ ($r$ is radial distance from centre of vortex), concluding at some outer boundary $r=R$ to avoid fluid particles ...
Si Chen's user avatar
  • 405
2 votes
1 answer
195 views

How do the Navier-Stokes equations predict vortex creation in a vorticity-free fluid?

For this question I was looking at the Navier-Stokes equations after you get the curl, this gives you the equation: $$\frac{\partial \vec{\Omega}}{\partial t}-\nabla \times (\vec{v} \times \vec{\...
Physics_Boss_India's user avatar
2 votes
2 answers
392 views

Nomenclature clarification concerning solitons

My experience with solitons is restricted to the classical setting, namely solutions to the quartic interaction $\phi^4$, the Sine-Gordon equation, and Korteweg–de Vries equations. I was explicit to ...
sunspots's user avatar
  • 702
2 votes
2 answers
3k views

The relationship between height of water and the rotational speed inside a cup

When I stir the water inside a cup with a speed $v$ ( using a stick), I find that a vortex will be formed inside the cup, with the middle part of the water at the lowest point ( with the height $h_0$),...
Graviton's user avatar
  • 2,536
2 votes
0 answers
163 views

The Optics of the Vortex (in Water): why there is a bright ring also in reversed flow?

This question is closely related to this one; The optics of vortex (in water): why there is a bright ring? In this case "physics girl" gives somehow an plausible explanation, and it's only debatable ...
Jokela's user avatar
  • 2,445
1 vote
2 answers
256 views

vortex anti-vortex configuration to ground state [closed]

i'm studying kosterlitz transition. I'm reading this: https://assets.nobelprize.org/uploads/2018/06/advanced-physicsprize2016-1.pdf?_ga=2.51324009.1302372948.1538119052-1605759177.1538119052 Now ...
MementoMori's user avatar
1 vote
0 answers
144 views

What causes a whirlpool to form in a bathtub? [duplicate]

I mean, when you have a full bathtub and then let it empty, under what conditions does a whirlpool form? Could we devise an experiment and measure this effect? When it comes to fluids, I don't know ...
Alubeixu's user avatar
  • 282
1 vote
1 answer
142 views

How do the vortices in vortex-induced vibrations (VIV) form?

I am currently doing a research project on the topic of the Vortex Bladeless technology in a technical college course. I have been researching VIV and the intuition behind this, but I am still ...
Jacob Bunzel's user avatar
1 vote
1 answer
229 views

Explanation for the next steps of chaplygin dipole

this post is the Chaplygin dipole, it's an interesting issue. Can someone explain me these steps in other words please? any Explanation of any step will help me, I hope that together I will ...
Alon Shmiel's user avatar
1 vote
0 answers
246 views

When two opposing vortexes combine to form a dipole vortex, why does it always move in the same direction?

AS you know, when two opposing vortexes combine to form a dipole vortex, it always move in the same direction as shown with red arrow in figure below. I wonder, why? is there a equation for this ...
Hakan Egne's user avatar
0 votes
1 answer
67 views

Metric of a rotating Cosmic String

I searched on the internet superficially but I couldn't find it. Is there any reference that find the solution to Einstein's Field Equations for a rotating Cosmic String? Personally haven't got the ...
0 votes
1 answer
327 views

Energy of vortex-antivortex pair

I am given the following field in 2d: $$\theta(\mathbf{x}) = \phi(\mathbf{x}-\mathbf{x}_1) - \phi(\mathbf{x}-\mathbf{x}_2),$$ where $\mathbf{x}_1,\mathbf{x}_2$ are the position of the vortex and ...
Mathphys meister's user avatar
0 votes
2 answers
803 views

Can one model a spiral galaxy with a similar manifold to one for a whirlpool?

My question is more of a phenomenological question than literal, since the forces involved are very different. When one looks at the distortions (or wrinkles or whatever you wish to call them) in the ...
honeste_vivere's user avatar